Q22

 
angela
Thanks Received: 8
LSAT Geek
 
Posts: 17
Joined: April 07th, 2009
 
 
 

Q22

by angela Wed Dec 07, 2011 5:02 pm

Please take a look at the diagram thread to read a full explanation for this game. If you have any follow-up questions about this particular question, please reply in this thread so we keep things tidy! Thank you.

diagram-t211.html
 
timmydoeslsat
Thanks Received: 887
Atticus Finch
Atticus Finch
 
Posts: 1136
Joined: June 20th, 2011
 
 
trophy
Most Thanked
trophy
First Responder
 

Re: Q22

by timmydoeslsat Sat Jan 21, 2012 11:30 pm

I got this question right by process of elimination, but is answer choice C truly a complete list?

Could you not also include Friday? If you could include Friday, then answer C could not be a complete list.


M: 10 Z
T: 10 Z
W: 12 Y
R: 15 X
F: 10 Z

All of the rules are met. More Z's than X's. Thursday is the 15 dollar lot. X costs more than Z. Friday looks to be a valid day for the inclusion of another 10 dollar lot. What am I missing?
 
timmydoeslsat
Thanks Received: 887
Atticus Finch
Atticus Finch
 
Posts: 1136
Joined: June 20th, 2011
 
 
trophy
Most Thanked
trophy
First Responder
 

Re: Q22

by timmydoeslsat Mon Jan 23, 2012 1:55 pm

I am going to leave my initial post up even though I am going to answer my own post!

This question is asking what could be a complete and accurate list of the days.

It is not asking what is a complete and accurate list of the days. If this were the question, Friday would be required.

However, with the question being what could be a complete and accurate list of days, answer C is correct.

What is interesting about this question is that a strong grasp of interchangeability can lead to an answer in a hurry.

We can eliminate answer choices D and E simply by looking at a global diagram. The first rule tells us that Thursday will not be a place for the 10 lot. We also know that rule 3 tells us that Wednesday could never be a 10 lot. If it were a 10 lot, how would it be more expensive than Friday?

So we have

A) Monday
B) Tuesday
C) Monday, Tuesday

Since the rules only govern the # of times a variable goes and which lots cost more than the other...Monday and Tuesday have no rules governing their use.

There is not an ordering aspect that would nullify their interchangeability. If Monday could be true, then Tuesday could be true, and vice versa. Thus, you know that C has to be the answer.
User avatar
 
ManhattanPrepLSAT1
Thanks Received: 1909
Atticus Finch
Atticus Finch
 
Posts: 2851
Joined: October 07th, 2009
 
This post thanked 1 time.
 
 

Re: Q22

by ManhattanPrepLSAT1 Wed Jan 25, 2012 8:22 pm

Totally agree that you can "guess" the correct answer fairly easy on this one.

My line of reasoning on ran that since the question was looking for what could be a complete and accurate list of the days and not for what is a complete and accurate list of the days she could park in the $10 lot and Friday wasn't included in the answers, the Friday would have to be the $12 lot. If Friday is the $12 lot, Wednesday is the $15 lot. She can't park in X on more than one day, so the $15 lot had to be Y. At this point we've got Y ($15) on Wed + Thur and X ($12) on Fri. We need more Z's than X's, so Z would have to go Monday and Tuesday; that's how you know that answer choice (A) or (B) couldn't also be a complete and accurate list.

I found the two distribution tables very useful on this game.

Cost / Lot
X -- Y -- Z
12 - 15 - 10
15 - 12 - 10
15 - 10 - 12

# of days/Lot
X - Y - Z
1 - 2 - 2
1 - 1 - 3
 
hilarykustoff
Thanks Received: 0
Jackie Chiles
Jackie Chiles
 
Posts: 31
Joined: November 15th, 2011
 
 
 

Re: Q22

by hilarykustoff Fri Mar 23, 2012 4:57 pm

I'm totally confused on this question? I don't even know where to begin on this one. Thanks for your help!
 
timmydoeslsat
Thanks Received: 887
Atticus Finch
Atticus Finch
 
Posts: 1136
Joined: June 20th, 2011
 
 
trophy
Most Thanked
trophy
First Responder
 

Re: Q22

by timmydoeslsat Fri Mar 23, 2012 9:11 pm

We commonly see questions on the logic games about complete and accurate lists.

However, what may not catch eye but absolutely must, is the distinction between "is a complete and accurate list" and "could be a complete and accurate list."

I will define each.

Is a complete and accurate list:
This is literally asking you for a laundry list of items that can appear in a certain manner. This is not asking for a single hypothetical. This can have as many valid hypotheticals as needed to come up with this list.


Could be a complete and accurate list:
This is literally asking for a hypothetical that is valid. When you see those first questions that usually come up on logic games where you apply the rules one by one, that is a "could be a complete and accurate list." That is not an actual laundry list of, say, items that can go fifth in a game.

As for this question, this is asking us the latter type, the could be a complete and accurate list of the days she can park in the $10 lot.

We know from our setup that Monday and Tuesday have no information about them. They are interchangeable with each other as there is not an actual ordering aspect, such as she must park in this lot before that lot. Since Monday and Tuesday are interchangeable, they are equals. If choice A were true, than so would choice B---and vice versa. Since we know that there can only be one correct answer, we can validly eliminate both A and B due to our knowledge of interchangeability.

We are now left with C, D, and E. We can eliminate E since it has Thursday on there. We know from the set up that Thursday is a $15 lot, so it cannot also be a $10 lot.

We are left with C and D. We know that Wednesday cannot be a $10 lot because Wednesday must be more than Friday in cost. So this eliminates D. We are left with C.
 
stacksdoe
Thanks Received: 0
Forum Guests
 
Posts: 54
Joined: August 19th, 2012
 
 
 

Re: Q22

by stacksdoe Tue May 12, 2015 2:25 am

hilarykustoff-
I'm totally confused on this question? I don't even know where to begin on this one. Thanks for your help!

The individual who provided the summary between "a complete and accurate list" compared to a "could be a complete and accurate list" captured the key difference: one question stem wants a complete and accurate list and another wants a hypothetical case for a complete and accurate list. To put it another way, one question asks for what must be the case (the former) and the other asks for what could be the case (the latter). Consequently, a question that asks for "a complete and accurate list" is technically asking for what must be true while masquerading as a "what could be true" question. So,as Timmy articulated, if this question was asking for "a complete and accurate list" instead of "what could be…." then the correct answer would have to include ALL the possible days she could park in the $10 lot. If this seems confusing, message me and hopefully I can better explain.
Back to your question about where to begin:
First and foremost, if your unsure where to begin, try to eliminate some answer choices. Since you know you are looking for days that could be $10, ask yourself are there any days we know from the initial set up that can never be $10 lots? Thursday has to always be $15, so eliminate (E), Wednesday can only be 12/15 in all cases, so eliminate (D). Now your left with only three answer choices.
Like in many other cases, there are a few ways to arrive at the correct solution.
Because your left with either a one day possibility or a two day possibility, the best thing to do is asses which variable can be slotted for more then one day and construct a hypothetical case (mini diagram). One option is this:
We know that its between Y and Z, because X can not be a $10 lot. And because they're are no restrictions on Monday or Tuesday it's highly likely what can happen on Monday can also happen on Tuesday (the rule of interchangeability). Thus if you can prove that Y or Z can be slotted just for Monday, or just for Tuesday, or both, then your done. I won't go into detail, just remember that X can only be placed on one day, Y can be slotted for one or two days, and Z can be slotted for two or three days. No matter how you sketch your mini diagram or arrive at the solution, it will always be (for a complete list): Monday and Tuesday, or, either Monday or Tuesday along with Friday, or just Friday. And once I noticed that Friday was missing from the answer choices, I realized what I missed in the question stem, the question is asking for a "could be" a complete list instead of what "must be" a complete list.
A good place to start sketching is slot Monday or Tuesday with $10 lot and give it Y or Z and try to finish the diagram, in all cases you'll find that the only possible answer choice available is C. A side note, had they included Friday alone as an option, that would also be correct. Interestingly enough, they completely excluded Friday.
Also remember that the last question is generally the harder question and thus calls for a little more detail work.
I hope that this helps instead of adding more confusion.

Stacks